Inverse of the matrix of a linear transformation

Click For Summary
SUMMARY

The discussion focuses on determining the invertibility of the linear transformation T: M22→M22 defined by T(A)=AB, where B is the matrix [[3,2],[2,1]]. The user attempts to find T^-1 and presents their calculations, which include T and T^-1 matrices. The conclusion drawn is that to ascertain the invertibility of T, it is essential to check if matrix B has an inverse, as this directly impacts the invertibility of the transformation.

PREREQUISITES
  • Understanding of linear transformations in matrix algebra
  • Knowledge of matrix inversion and determinants
  • Familiarity with standard bases in linear algebra
  • Proficiency in manipulating matrices and performing matrix multiplication
NEXT STEPS
  • Research the conditions for matrix invertibility, specifically for 2x2 matrices
  • Learn how to compute the inverse of a matrix using the formula for 2x2 matrices
  • Study the implications of linear transformations on vector spaces
  • Explore the relationship between linear transformations and their matrix representations
USEFUL FOR

Students studying linear algebra, particularly those focusing on linear transformations and matrix theory, as well as educators seeking to clarify concepts related to matrix invertibility.

alias
Messages
40
Reaction score
0

Homework Statement


Let T: M22→M22 be a LT defined by T(A)=AB where
B=[3,2
2,1]
Determine if T is invertible with respect to standard bases B=C={e11,e12,e21,e22}. If so, use (equation below) to find T^-1.

Homework Equations



[[T^-1 [AB]]C = [T^-1]B to C matrix [AB]B (at least I think this is the right one)

The Attempt at a Solution



I found:
T= [ [3,2,0,0]’,[2,1,0,0]’,[0,0,3,2]’,[0,0,2,1]’ ]
T^-1 = [ [-1,2,0,0]’,[2,-3,0,0]’,[0,0,-1,2]’,[0,0,2,-3]’ ]
My TA told me I was doing this wrong and I can't figure out what to do. Can anyone help?
 
Last edited:
Physics news on Phys.org
so if get it right, if T is invertible then
T^{-1}(AB) = (AB)B^{-1} = A

so isn't it sufficent to find whether B has an inverse
 
Much appreciated
 
Question: A clock's minute hand has length 4 and its hour hand has length 3. What is the distance between the tips at the moment when it is increasing most rapidly?(Putnam Exam Question) Answer: Making assumption that both the hands moves at constant angular velocities, the answer is ## \sqrt{7} .## But don't you think this assumption is somewhat doubtful and wrong?

Similar threads

  • · Replies 18 ·
Replies
18
Views
3K
  • · Replies 2 ·
Replies
2
Views
3K
  • · Replies 8 ·
Replies
8
Views
2K
  • · Replies 3 ·
Replies
3
Views
2K
  • · Replies 5 ·
Replies
5
Views
2K
  • · Replies 1 ·
Replies
1
Views
2K
  • · Replies 4 ·
Replies
4
Views
2K
  • · Replies 4 ·
Replies
4
Views
2K
  • · Replies 6 ·
Replies
6
Views
2K
  • · Replies 11 ·
Replies
11
Views
2K